A particular state's license plates have 7 characters. Each character can be a capital letter, or a digit except for 0. How many license plates are there in which no two adjacent characters are the same

Answers

Answer 1

Answer:

N = 35 × 34^6

N = 54,068,154,560

Step-by-step explanation:

Given;

The license plate have 7 characters.

Each character can be a capital letter, or a digit except for 0.

There are 26 capital letters

And there are 9 digits excluding 0

The total number of possible entries in each character is;

26+9 = 35

The number of license plates in which no two adjacent characters are the same are;

For no two adjacent characters of the license plate not to be the same that is no two characters that follow each other can be the same, the number of possible entries in the adjacent characters apart from the first character would be reduced by one.

N = 35 × 34 ×34×34×34×34×34

N = 35 × 34^6

N = 54,068,154,560

Therefore, there are 54,068,154,560 possible license plates


Related Questions

You are shopping online for a new TV. The one you want costs $2,500. You live in New York where the state sales tax rate is 8.49%. Your sister lives a short distance over the border in CT where the state sales tax rate is 6.35%. If you have the TV shipped to her and you pick it up. How much will you save?

Answers

Answer:

$5350

Step-by-step explanation:

see attachment

Answer:

Its 53.50

Step-by-step explanation:

Solve the inequality 6h−5(h−1)≤7h−11 and write the solution in interval notation. Use improper fractions if necessary.

Answers

Answer:

[tex]h \geq 2\frac{2}[3}[/tex]

Step-by-step explanation:

We solve the inequality similarly to how we would solve an equalitu.

[tex]6h - 5(h-1) \leq 7h - 11[/tex]

[tex]6h - 5h + 5 \leq 7h - 11[/tex]

[tex]h - 7h \leq -11 - 5/[/tex]

[tex]-6h \leq -16[/tex]

Multiplying everything by -1

[tex]6h \geq 16[/tex]

Simplifying by 2

[tex]3h \geq 8[/tex]

[tex]h \geq \frac{8}{3}[/tex]

8 divided by 3 is 2 with rest two. So as a improper fraction, the answer is:

[tex]h \geq 2\frac{2}[3}[/tex]

Answer:

[8,

3

Step-by-step explanation:

Rule multiply the last number by 3 then subtract 2

2 4 10 _ _

Answers

The answer is 28, 82

Answer:

[tex]28[/tex], [tex]82[/tex]

Step-by-step explanation:

[tex]10 \times 3 -2=28\\28 \times 3 - 2 = 82[/tex]

The tree diagram represents an
experiment consisting of two trials.
.4
.5
.6
D
.3
С
.5
.7
D
Þ(B and D) = [?]
ter

Answers

Answer:

0.35

Step-by-step explanation:

.5x.7

Tara has 1 3/5

yards of fabric. She needs

2 1/2

times this amount to make a shopping bag. How much fabric does Tara need to make the bag?

Answers

Answer:

amount of fabric to make the bag = 4 yards

Step-by-step explanation:

Tara has 1 3/5 yards of fabric . She needs extra 2 1/2 times the amount she have to make a shopping bag. The amount of fabric she needs to make the bag can be calculated as follows.

1 3/5 yards = 8/5 yards of fabrics

What she actually needs to make a shopping bag is two and half the amount she has . Mathematically, it can be express

2 1/2 × 8/5

Let us change 2 1/2 to improper fraction

amount of fabric to make the bag = 5/2 × 8/5

amount of fabric to make the bag = 40/10

amount of fabric to make the bag = 4 yards

find the circumference
(3.14)(24)

Answers

Answer:

75.36unit^2

Step-by-step explanation:

Looking at the expression in the question, it is synonymous to the

equation for the circumference of a circle

π×d =3.14×24=75.36unit^2

Diego's doctor has recommended that his daily diet should include 3 vegetables, 2 fruits, and 2 whole grains. At the grocery store, Diego has a choice of 9 vegetables, 9 fruits, and 11 whole grains. In how many ways can he get his daily requirements if he doesn't like to eat 2 servings of the same thing in 1 day

Answers

Answer:

166320 ways

Step-by-step explanation:

In this case we must calculate the number of combinations for each option, and then multiply the result of each one, like this:

Number of ways you can choose 3 vegetables out of 9 available:

nCr = n! / (r! * (n-r)!)

in this case n = 9, r = 3, replacing:

9C3 = 9! / (3! * (9-3)!) = 84

Number of ways you can choose 2 fruits out of 9 available:

9C2 = 9! / (2! * (9-2)!) = 36

Number of ways you can choose 2 whole grains out of 11 available:

11C2 = 11! / (2! * (11-2)!) = 55

So according to the rule of the products how many ways you can choose the daily diet

84*36*55 = 166320

What is the y value?

Answers

Answer:

y =34

Step-by-step explanation:

The sum of the angles of a triangle add to 180 degrees

63+ 2y+ y+15 = 180

Combine like terms

78+ 3y = 180

Subtract 78 from each side

78+3y-78 = 180-78

3y = 102

Divide each side by 3

3y/3 = 102/3

y =34

Answer:

[tex]y = 34[/tex]

Step-by-step explanation:

Sum of the angles in a triangle is 180 degrees

[tex]2y + 63 + y + 15 = 180 \\ 3y + 63 + 15= 180 \\ 3y +7 8 = 180 \\ 3y = 180 - 78 \\ 3y =10 2 \\ \frac{3y}{3} = \frac{102}{3} \\ y = 34 [/tex]

hope this helps

brainliest appreciated

good luck! have a nice day!

You measure 50 textbooks' weights, and find they have a mean weight of 37 ounces. Assume the population standard deviation is 5.2 ounces. Based on this, construct a 90% confidence interval for the true population mean textbook weight. Give your answers as decimals, to two places

Answers

Answer:

90% confidence interval for the true population mean textbook weight is [35.79 ounces , 38.21 ounces].

Step-by-step explanation:

We are given that you measure 50 textbooks' weights, and find they have a mean weight of 37 ounces.

Assume the population standard deviation is 5.2 ounces.

Firstly, the Pivotal quantity for 90% confidence interval for the population mean is given by;

                           P.Q. =  [tex]\frac{\bar X-\mu}{\frac{\sigma}{\sqrt{n} } }[/tex]  ~ N(0,1)

where, [tex]\bar X[/tex] = sample mean weight = 37 ounces

           [tex]\sigma[/tex] = population standard deviation = 5.2 ounces

           n = sample of textbooks = 50

           [tex]\mu[/tex] = true population mean textbook weight

Here for constructing 90% confidence interval we have used One-sample z test statistics as we know about population standard deviation.

So, 90% confidence interval for the population mean, [tex]\mu[/tex] is ;

P(-1.645 < N(0,1) < 1.645) = 0.90  {As the critical value of z at 5%

                                            level of significance are -1.645 & 1.645}  

P(-1.645 < [tex]\frac{\bar X-\mu}{\frac{\sigma}{\sqrt{n} } }[/tex] < 1.645) = 0.90

P( [tex]-1.645 \times {\frac{\sigma}{\sqrt{n} } }[/tex] < [tex]{\bar X-\mu}[/tex] < [tex]1.645 \times {\frac{\sigma}{\sqrt{n} } }[/tex] ) = 0.90

P( [tex]\bar X -1.645 \times {\frac{\sigma}{\sqrt{n} } }[/tex] < [tex]\mu[/tex] < [tex]\bar X +1.645 \times {\frac{\sigma}{\sqrt{n} } }[/tex] ) = 0.90

90% confidence interval for [tex]\mu[/tex] = [ [tex]\bar X -1.645 \times {\frac{\sigma}{\sqrt{n} } }[/tex] , [tex]\bar X +1.645 \times {\frac{\sigma}{\sqrt{n} } }[/tex] ]

                                          = [ [tex]37-1.645 \times {\frac{5.2}{\sqrt{50} } }[/tex] , [tex]37+1.645 \times {\frac{5.2}{\sqrt{50} } }[/tex] ]

                                          = [35.79 , 38.21]

Therefore, 90% confidence interval for the true population mean textbook weight is [35.79 ounces , 38.21 ounces].

The sum of two numbers is 41. One number is 1 less than the other.

Answers

Answer:

s must be 21 and r is 20

Step-by-step explanation:

Represent the two numbers using variables r and s.

Then r + s = 41, and r = s - 1

Substituting the 2nd equation for r into the first equation, we get:

s - 1 + s = 41, or 2s = 42.  Then s must be 21 and r is 20.

r(x−s)=sx+t what does x equal

Answers

Answer:

x = (rs + t)/(r-s)

Step-by-step explanation:

r(x - s) = sx + t

rx - rs = sx + t

rx - sx = rs + t

x(r - s) = rs + t

x = (rs + t)/(r-s)

The solution for x is x = (t + rs)/(r - s).

What is an equation?

A pair of algebraic equations with the equal symbol (=) in the center and the same value are referred to as an equation.

We can begin by simplifying the given equation:

First, solve the parenthesis,

r(x - s) = sx + t

Simplify, the equation, we get,

rx - rs = sx + t

rx - sx = t + rs

Take the like terms to one side, we get,

x(r - s) = t + rs

Now, solve for x, we get,

x = (t + rs)/(r - s)

Therefore, x is equal to (t + rs)/(r - s).

To learn more about the equation;

https://brainly.com/question/12788590

#SPJ7

A phone company charges 12 cents per minute of call. If Gerardo made a call that took 75 minutes using this plan, how much did his call cost?

Answers

Answer:

900 cents, 9 dollars

Step-by-step explanation:

Dan got 45 out of 72 correct in his test.
What fraction of the marks did he get wrong?
Give your answer in its simplest form.

Answers

Answer:

3/8

Step-by-step explanation:

45/72=5/8

(5/8)+x=72/72

x=1-(5/8)

x=3/8

What is the range of the function?
all real numbers less than or equal to 4
all real numbers less than or equal to -3
all real numbers greater than or equal to 4
all real numbers greater than or equal to -3
10X

Answers

Answer:

1. all real numbers less than or equal to 4 = </= 4.

Answer = YES!

2. all real numbers less than or equal to -3 = </= -3.

Answer = YES!

3. all real numbers greater than or equal to 4 = >/= 4

Answer = YES!

4. all real numbers greater than or equal to -3 = >/= -3

Answer = NO!

Step-by-step explanation:

1. all real numbers less than or equal to 4 = </= 4.

1 + 1 = 2

1 + 2 = 3

2 + 2 = 4

2. all real numbers less than or equal to -3 = </= -3.

1 - 1 = 0

1 - 2 = -1

1 - 3 = -2

1 - 4 = -3

3. all real numbers greater than or equal to 4

1 + 1 = 2

1 + 2 = 3

1 + 3 = 4

1 + 4 = 5

4. all real numbers greater than or equal to -3

1 - 1 = 0

1 - 2 = -1

1 - 3 = -2

1 - 4 = -3

Answer:

The range is all real numbers less than or equal to 4

Step-by-step explanation:

took the test on edge

If s(x) = 2 – x2 and t(x) = 3x, which value is equivalent to (s circle t) (negative 7)?

Answers

Answer:

153

Step-by-step explanation:

What is the value of y?

Answers

Answer:

d. 54

Step-by-step explanation:

Answer:

D.

Step-by-step explanation:

y=(180º-72º)/2

y=108º/2

y=54º

D.

What is the value of x in the equation 1.5(x + 4) - 3 = 4.5(x - 2)?
3
4
Ο Ο Ο Ο
5
9

Answers

Answer:

The answer is 4

Step-by-step explanation:

1.5(4+4)-3= 9

4.5(4-2)=9

9=9

Answer:

[tex]x=4[/tex]

Explanation:

[tex]1.5x+6+-3=4.5x+-9\\1.5x+3=4.5x-9\\-3x+3=-9\\-3x=-12\\x=4[/tex]

Twenty students from Sherman High School were accepted at Wallaby University. Of those students, eight were offered military scholarships and 12 were not. Mr. Dory believes Wallaby University may be accepting students with lower SAT scores if they have a military scholarship. The newly accepted student SAT scores are shown here.


Military scholarship: 850, 925, 980, 1080, 1200, 1220, 1240, 1300

No military scholarship: 820, 850, 980, 1010, 1020, 1080, 1100, 1120, 1120, 1200, 1220, 1330


Part A: Do these data provide convincing evidence of a difference in SAT scores between students with and without a military scholarship? Carry out an appropriate test at the α = 0.05 significance level.


Part B: Create and interpret a 95% confidence interval for the difference in SAT scores between students with and without a military scholarship.

Answers

Answer:

Step-by-step explanation:

For military scholarship,

Mean, x1 = (850 + 925 + 980 + 1080 + 1200 + 1220 + 1240 + 1300)/8

x1 = 1099.375

Standard deviation = √(summation(x - mean)²/n

n1 = 8

Summation(x - mean)² = (850 - 1099.375)^2 + (925 - 1099.375)^2 + (980 - 1099.375)^2 + (1080 - 1099.375)^2 + (1200 - 1099.375)^2 + (1220 - 1099.375)^2 + (1240 - 1099.375)^2 + (1300 - 1099.375)^2 = 191921.875

Standard deviation, s1 = √(191921.875/8

s1 = 154.89

For no military scholarship,

Mean, x2 = (820 + 850 + 980 + 1010 + 1020 + 1080 + 1100 + 1120 + 1120 + 1200 + 1220 + 1330)/12

x2 = 1070.83

Standard deviation = √(summation(x - mean)²/n

n2 = 12

Summation(x - mean)² = (820 - 1070.83)^2 + (850 - 1070.83)^2 + (980 - 1070.83)^2 + (1010 - 1070.83)^2 + (1020 - 1070.83)^2 + (1080 - 1070.83)^2 + (1100 - 1070.83)^2 + (1120 - 1070.83)^2 + (1120 - 1070.83)^2 + (1200 - 1070.83)^2 + (1220 - 1070.83)^2 + (1330 - 1070.83)^2 = 238091.6668

Standard deviation, s2 = =√(238091.6668/12

s2 = 140.86

Part A)

This is a test of 2 independent groups. The population standard deviations are not known. Let μ1 be the mean score of students with military scholarship and μ2 be the mean score of students without military scholarship.

The random variable is μ1 - μ2 = difference in the mean score between students with military scholarship and without military scholarship

We would set up the hypothesis.

The null hypothesis is

H0 : μ1 = μ2 H0 : μ1 - μ2 = 0

The alternative hypothesis is

H1 : μ1 ≠ μ2 H1 : μ1 - μ2 ≠ 0

This is a two tailed test

Since sample standard deviation is known, we would determine the test statistic by using the t test. The formula is

(x1 - x2)/√(s1²/n1 + s2²/n2)

t = (1099.375 - 1070.83)/√(154.89²/8 + 140.86²/12)

t = 0.42

The formula for determining the degree of freedom is

df = [s1²/n1 + s2²/n2]²/(1/n1 - 1)(s1²/n1)² + (1/n2 - 1)(s2²/n2)²

df = [154.89²/8 + 140.86²/12]²/[(1/8 - 1)(154.89²/8)² + (1/12 - 1)(140.86²/12)²] = 21644133.914878543/1533280.3458504018

df = 14

We would determine the probability value from the t test calculator. It becomes

p value = 0.68

Since alpha, 0.05 < than the p value, 0.68, then we would fail to reject the null hypothesis. Therefore, at a significance level of 5%, these data do not provide convincing evidence of a difference in SAT scores between students with and without a military scholarship

Part B)

The formula for determining the confidence interval for the difference of two population means is expressed as

Confidence interval = (x1 - x2) ± z√(s²/n1 + s2²/n2)

For a 95% confidence interval, we would determine the z score from the t distribution table because the number of samples are small

Degree of freedom =

(n1 - 1) + (n2 - 1) = (8 - 1) + (12 - 1) = 18

z = 2.101

x1 - x2 = 1099.375 - 1070.83 = 28.545

Margin of error = 2.101√(154.89²/8 + 140.86²/12) = 143.3

The 95% confidence interval is 28.545 ± 143.3

Which expression is equivalent to StartRoot negative 80 EndRoot? Negative 4 StartRoot 5 EndRoot Negative 4 StartRoot 5 EndRoot i 4 StartRoot 5 EndRoot i 4 StartRoot 5 EndRoot


or

Which expression is equivalent to √-80


A. -4√5

B. -4√5i

C. 4√5i

D. 4√5

Answers

The answer is D. 4√5

Answer:

C. 4√5i

Step-by-step explanation:

on edge

please vote brainliest i have never gotten it before

What’s the correct answer for this?

Answers

Answer:

B

Step-by-step explanation:

OP = OQ

Answer:

B

Step-by-step explanation:

Look at the shape closely observe the following the imaginary lines;

OC = OA , OB = OD {these is the raduis of the circle;

note also line CQ = CD { the perpendicular line bisects line CD in two equal halves;

Similarly ;

note also line AP= PB { the perpendicular line bisects line OP in two equal halves}

Hence;

OP is congruent to the OQ

In maths we say something is congruent when they are the same shape and have same angles but we are allowed to flip it's side.

Help ill give you brainliest and points !!!!!

Answers

Answer:

1.12

2.85

3.60

Step-by-step explanation:

1.

if m = 12 you solve by dividing 144/12

144/12 = 12

2.

when m = 11 you solve by computing 7(11) + 8

7(11) = 77 so you solve from there 77 + 8 which equals 85

3.

When g = 4 you solve my multiplying 15(4)

15(4) = 60

Answer:

first question: 12

second question: 85

third question: 60

Step-by-step explanation:

first question: plug in 12 for m and you would get 144/12 which equals 12.

Second question: plug in 11 for m and you have 7(11) + 8.

77+8 = 85

third question: plug in 4 for g and you have 15(4)

15 x 4 = 60.

*parentheses mean multiplication when next to a number

ex: 7(11)

not ex: 3 + (2/3) since it isn't directly next to the first number, instead its adding

If AB=x and x=4, then the transitive property states


1. AB+4=2x

2. X=4

3. AB+x=4

4. AB=4

Answers

Answer:

5

Step-by-step explanation:

i took the test

The transitive property states AB = 4.

What is transitive property?

The transitive property of equality states that the first number is also equal to the third number if two numbers are equal and the second number is equal to the third number. In other words, if a is equal to b and b is equal to c, then a is equal to c. One of the many mathematical properties of equality is the transitive property.

Given AB = x and x = 4

Acc. to transitive property,

if a = b, b = c Then c = a.

so AB = 4

Hence option D is correct, AB = 4.

Learn more about transitive property;

https://brainly.com/question/2437149

#SPJ5

Of all the companies on the New York Stock Exchange, profits are normally distributed with a mean of $6.54 million and a standard deviation of $10.45 million. In a random sample of 73 companies from the NYSE, what is the probability that the mean profit for the sample was between 0 million and 5.1 million?

Answers

Answer:

11.90% probability that the mean profit for the sample was between 0 million and 5.1 million

Step-by-step explanation:

To solve this question, we need to understand the normal probability distribution and the central limit theorem.

Normal probability distribution

Problems of normally distributed samples are solved using the z-score formula.

In a set with mean [tex]\mu[/tex] and standard deviation [tex]\sigma[/tex], the zscore of a measure X is given by:

[tex]Z = \frac{X - \mu}{\sigma}[/tex]

The Z-score measures how many standard deviations the measure is from the mean. After finding the Z-score, we look at the z-score table and find the p-value associated with this z-score. This p-value is the probability that the value of the measure is smaller than X, that is, the percentile of X. Subtracting 1 by the pvalue, we get the probability that the value of the measure is greater than X.

Central Limit Theorem

The Central Limit Theorem estabilishes that, for a normally distributed random variable X, with mean [tex]\mu[/tex] and standard deviation [tex]\sigma[/tex], the sampling distribution of the sample means with size n can be approximated to a normal distribution with mean [tex]\mu[/tex] and standard deviation [tex]s = \frac{\sigma}{\sqrt{n}}[/tex].

For a skewed variable, the Central Limit Theorem can also be applied, as long as n is at least 30.

In this question, we have that:

[tex]\mu = 6.54, \sigma = 10.45, n = 73, s = \frac{10.45}{\sqrt{73}} = 1.2231[/tex]

In a random sample of 73 companies from the NYSE, what is the probability that the mean profit for the sample was between 0 million and 5.1 million?

This is the pvalue of Z when X = 5.1 subtracted by the pvalue of Z when X = 0. So

X = 5.1

[tex]Z = \frac{X - \mu}{\sigma}[/tex]

By the Central Limit Theorem

[tex]Z = \frac{X - \mu}{s}[/tex]

[tex]Z = \frac{5.1 - 6.54}{1.2231}[/tex]

[tex]Z = -1.18[/tex]

[tex]Z = -1.18[/tex] has a pvalue of 0.1190

X = 0

[tex]Z = \frac{X - \mu}{s}[/tex]

[tex]Z = \frac{0 - 6.54}{1.2231}[/tex]

[tex]Z = -5.35[/tex]

[tex]Z = -5.35[/tex] has a pvalue of 0

0.1190 - 0 = 0.1190

11.90% probability that the mean profit for the sample was between 0 million and 5.1 million

George is curious. He has been told that the average body temperature of humans is 98.6 degrees Fahrenheit.​ However, he believes it is much lower than that. He randomly selects 60 people from those passing by him on a street and takes their temperature. The average temperature of these 60 people is 98.2 degrees Fahrenheit. The standard​ deviation, sigma Subscript x​,is known to be 0.62 degrees Fahrenheit. The​ p-value is less than 0.0001. What is the correct​ conclusion?

Answers

Answer:

[tex]p_v =P(t_{(59)}>t_{calc})<0.0001[/tex]  

Since the p value is a very low value we have enough evidence to reject the null hypothesis in favor of the alternative hypothesis and we can conclude that the true mean for this case is significantly different from 98.6 F at any usual significance level used.

Step-by-step explanation:

Information given

[tex]\bar X=98.2[/tex] represent the sample mean

[tex]\sigma=0.62[/tex] represent the population standard deviation

[tex]n=60[/tex] sample size  

[tex]\mu_o =98.6[/tex] represent the value that we want to test

t would represent the statistic

[tex]p_v[/tex] represent the p value

System of hypothesis

We want to verify if the true mean is equal to 98.6 F, the system of hypothesis would be:  

Null hypothesis:[tex]\mu = 98.6[/tex]  

Alternative hypothesis:[tex]\mu \neq 98.6[/tex]  

The statistic is given by:

[tex]t=\frac{\bar X-\mu_o}{\frac{s}{\sqrt{n}}}[/tex]  (1)  

The degrees of freedom, on this case:  

[tex]df=n-1=60-1=59[/tex]  

The p value would be given by:

[tex]p_v =P(t_{(59)}>t_{calc})<0.0001[/tex]  

Since the p value is a very low value we have enough evidence to reject the null hypothesis in favor of the alternative hypothesis and we can conclude that the true mean for this case is significantly different from 98.6 F at any usual significance level used.

9 is .03% of what number?

Answers

Answer:

30,000

Step-by-step explanation:

We know that divide the percentage by 100. After that you get the decimal thing then all you do is multiply the number and you get your answer that is  30,000.

Answer: 30,000

Please mark brainliest

Hope this helps.

Please answer this correctly

Answers

72+72+36= 144+36= 180

Answer:

Area of the figure = 176 m²

Step-by-step explanation:

Area of Rectangle = Length × Width

Dividing the whole figure

Rectangle 1:

6 × 18 = 108 m²

Rectangle 2 :

10 × 6 = 60 m²

Rectangle 3:

2 × 4 = 8 m²

Adding All

Area of the figure = 108 + 60 + 8

Area of the figure = 176 m²

Brainliest to whoever gets this correct Which of the following is not a polynomial identity?

Answers

Answer:

Hi there!

The correct answer is C.

Step-by-step explanation:

A polynomial identity  are equations that are true for all possible values of the variable. For example, x²+2x+1=(x+1)² is an identity.

Can someone plz help me solved this problem! I’m giving you 10 points! I need help plz help me! Will mark you as brainiest!

Answers

Answer:

domain: (-∞,∞)

range [-3,∞)

Step-by-step explanation:

The domain is the values that x can take

X can be any value so the domain is all real numbers

The range is the values that y can take

The minimum value is -3

The range is y ≥ -3

Suppose you have $200,000 in a bank term account. You earn 5% interest per annum from his account

Answers

Answer:

$190,000

Step-by-step explanation:

discount = x

original price = $200,000

discount% = 5%

x/200,000 = 5/100

x · 100 = 5 · 200,000

100x = 1,000,000

100x/100 = 1,000,000/100

x = 10,000

Sale price: $200,000 - $10,000 = $190,000

2.1.40: The police records of a city contain the following statistics on offenses for the month of May: 430 assaults, 146 robberies, and 131 drug sales. The records also show that 26 people were involved in both assault and robbery and that 33 people were involved in both assault and drug sales. What is the greatest possible number of offenders for May who satisfy these statistics

Answers

Answer:

648

Step-by-step explanation:

Number of Assaults, n(A)=430

Number of Robberies, n(R)=146

Number of Drug Sales Offenses, n(D)=131

Number involved in both assault and robbery= [tex]n(A\cap R)=26[/tex]

Number involved in both assault and drug sales= [tex]n(A\cap D)=33[/tex]

Therefore, to find the greatest possible number of offenders for May who satisfy these statistics, we remove the number of those who were involved in more than one crime.

This gives us:

[tex]=n(A)+n(R)+n(D)-n(A\cap R)-n(A\cap D)\\=430+146+131-26-33\\=648[/tex]

The greatest possible numbers of offenders in May is 648.

Other Questions
A square has a perimeter of 44cm. Calculate the area of the square. The most relevant aka correct answer will get brainiest :) 1. Solve the following equation forx. (3 points)-3x - 2 = -20X= The radius of a right circular cone is increasing at a rate of 1.6 in/s while its height is decreasing at a rate of 2.2 in/s. At what rate is the volume of the cone changing when the radius is 135 in. and the height is 135 in. PLEASE HELP GIVING BRAINLIEST TO ONLY CORRECT ANSWER which words could replace the subordinating conjunction in the sentence while maintaining its original structure Which equations could you solve using the addition property of equality? Check all that apply. 4 + y = 100 y 4 = 100 4y = 100 4 + 100 = y 100 = y 5 All of the following are true regarding ethics except: Multiple Choice Ethics are beliefs that distinguish right from wrong. Ethics rules are often set for CPAs. Ethics do not affect the operations or outcome of a company. Are critical in accounting. Ethics can be difficult to apply. // Please Answer // !!! Which answer describes one plant and one animal that is likely to be found in the tundra?a.) a plant with leathery or waxy leaves and an animal with a layer of fatb.) a plant with bright-colored flowers and an animal with sharp teethc.) a plant with broad, flat leaves and an animal with a fur coatd.) a plant with long roots and an animal with bright coloration There are 32 students in Mrs. Nyer's class. This is 8% of the students in theschool. How many students are in the school? 16. Simplify 4+ (-3) - 2x (-6)O One of the goals of the public health system is to inform, educate andempower people about health issues. Why is this an important aspect ofpublic health?O A. It's an important aspect of public health because empowered andeducated people are likely to take care of themselves and seekpreventative care.O B. It's an important aspect of public health because empowered andeducated people will know to avoid emergency care because it'sso expensive.C. It's an important aspect of public health because theseempowered and educated people will eventually becomeemployed in the public health system.D. It's an important aspect of public health because empowered andeducated people rarely require serious medical care. Evaluate for x = 2. (12x + 8)/4 Does anyone know how to photoshop a pic that will look legit Question of the dayFill in the table of values below for the equation y=x+3 WILL MARK BRAINIEST ASAP PLEASE HELP!!!!!!!!!!!!!!11111 What is the coefficient of the c-term of the algebraic expression 14 a minus 72 r minus c minus 34 d? -34 -1 0 1 What phone numbers should you keep next to your phone in addition to 911?a.your doctorc.Urgent Careb.your dentistd.all of the above When solving the equation 3x + 2 = 13 , Joseph used a property of algebra to transform the equation into 3x = 11 . Which property did he use ? A) Multiplication |B) Division C) AdditionD) Subtraction Can someone describe the industrial revolution? Australia is a major world producer of all of the following except __________. A. diamonds B. geothermal energy C. lead D. opals Please select the best answer from the choices provided A B C D (1)As a cloud of gas and dust collapses on itself and becomes more dense, the temperature at the center of the cloudA, IncreaseB, Decrease,C, Stay the same.(2)What is the first step in the nuclear fusion process?A, two helium atoms combine to make one hydrogen atomB, two hydrogen atoms combine to make one helium atomC, two helium atoms combine to make one carbon atom